Question

Problem 2 [10 pts] For a fixed matrices B, C e R2x2, let W = {1 € R2x2 | A*B=24*C}. Determine if W is a subspace of R2x2 (eit

0 0
Add a comment Improve this question Transcribed image text
Answer #1

Solution: given that {A 61 px? A&B = 20x ch. whese B and c are fined matrices. ana Win a non-empty subpet. O EW. 61 n=0 12 OXlet KEIR. Now, we need to show KP EW fon (KP) x B = K PAB - K(P&B) = k ( 2px() & kptc = 2(KPXC) = 2 (KP) ( a KPEW Hence, w is

Add a comment
Know the answer?
Add Answer to:
Problem 2 [10 pts] For a fixed matrices B, C e R2x2, let W = {1...
Your Answer:

Post as a guest

Your Name:

What's your source?

Earn Coins

Coins can be redeemed for fabulous gifts.

Not the answer you're looking for? Ask your own homework help question. Our experts will answer your question WITHIN MINUTES for Free.
Similar Homework Help Questions
  • Problem 2 (10 pts) For a fixed matrices B, C e R2x2, let W = {A...

    Problem 2 (10 pts) For a fixed matrices B, C e R2x2, let W = {A € R2x2 | A * B = 2A *C}. Determine if W is a subspace of R2x2 (either prove that it is, or show via specific counterexample that it isn't).

  • 4. Consider R2x2 with inner product (A, B) tr(ATB), and let V CR2x2 be the subspace...

    4. Consider R2x2 with inner product (A, B) tr(ATB), and let V CR2x2 be the subspace 1 1 1 0 This is consisting of upper-triangular matrices. Let B= a basis for V. (You do not need to prove this.) (a) (8 points) Use the Gram-Schmidt procedure on 3 to find an orthonormal basis for V. Find projy (B) (b) (4 points) Let B= 4. Consider R2x2 with inner product (A, B) tr(ATB), and let V CR2x2 be the subspace 1...

  • Problem 4. Let n E N, and let V be an n-dimensional vector space. Let(, ,):...

    Problem 4. Let n E N, and let V be an n-dimensional vector space. Let(, ,): V × V → R be an nner product on V (a) Prove that there exists an isomorphism T: V -R" such that (b) Is the isomorphism T you found in part (a) unique? Give a proof or a counterexample. (c) Let A be an n × n symmetric matrix such that T A > 0 for all nonzero ERT. Show that there exists...

  • Let W be the set of singular (noninvertible) matrices of order 2. Show that W is...

    Let W be the set of singular (noninvertible) matrices of order 2. Show that W is not a subspace of M2×2 with the standard matrix operations. Q1: Let W be the set of singular (noninvertible) matrices of order 2. Show that W is not a subspace of M2x2 with the standard matrix operations.-

  • Help on Questions 1-3 Math 311 Orthogonal & Symmetric Matrix Proofs 1. Let the n x...

    Help on Questions 1-3 Math 311 Orthogonal & Symmetric Matrix Proofs 1. Let the n x n matrices A and B be orthogonal. Prove that the sum A + B is orthogonal, or provide counterexample to show it isn't 2. Let the n x n matrix A be orthogonal. Prove A is invertible and the inverse A-1 is orthogonal, or provide a counterexample to show it isn't. 3. Suppose A is an n x n matrix. Prove that A +...

  • Problem 3. Let V and W be vector spaces, let T : V -> W be...

    Problem 3. Let V and W be vector spaces, let T : V -> W be a linear transformation, and suppose U is a subspace of W (a) Recall that the inverse image of U under T is the set T-1 U] := {VE V : T(v) E U). Prove that T-[U] is a subspace of V (b) Show that U nim(T) is a subspace of W, and then without using the Rank-Nullity Theorem, prove that dim(T-1[U]) = dim(Unin (T))...

  • 5 3 1 0 Problem 10 Let wi = ,W2 W3 Let W = Span{W1,W2, W3}...

    5 3 1 0 Problem 10 Let wi = ,W2 W3 Let W = Span{W1,W2, W3} C R6. 11 9 1 2 a) [6 pts] Use the Gram-Schmit algorithm to find an orthogonal basis for W. You should explicitly show each step of your calculation. 10 -7 11 b) [5 pts) Let v = Compute the projection prw(v) of v onto the subspace W using the 5 orthogonal basis in a). c) (4 pts] Use the computation in b) to...

  • -9 2. Let Vi-8.V2,andvs-2, let B -(V,V2,Vs), and let W be the subspace spanned , let B -(Vi,V2,V3...

    -9 2. Let Vi-8.V2,andvs-2, let B -(V,V2,Vs), and let W be the subspace spanned , let B -(Vi,V2,V3), and let W be the subspace spanned by B. Note that B is an orthogonal set. 17 a. 1 point] Find the coordinates of uwith respect to B, without inverting any matrices or L-2 solving any systems of linear equations. 35 16 25 b. 1 point Find the orthogonal projection of to W, without inverting any matrices or solving any systems of...

  • [B] Let W be the subspace of M22 given in problem [A] . (B.1) Show that...

    [B] Let W be the subspace of M22 given in problem [A] . (B.1) Show that the following set forms a basis for W: S = -5 (B.2) Obtain the coordinate vector for A = 3 relative to S. That is, find (A)s. -8 Show work! [B] Let W be the subspace of M22 given in problem [A] . (B.1) Show that the following set forms a basis for W: S = -5 (B.2) Obtain the coordinate vector for A...

  • 3. Let a, b, c E Z such that ca and (a,b) = 1. Show that...

    3. Let a, b, c E Z such that ca and (a,b) = 1. Show that (c, b) = 1. 4. Suppose a, b, c, d, e E Z such that e (a - b) and e| (c,d). Show that e (ad — bc). 5. Fix a, b E Z. Consider the statements P: (a, b) = 1, and Q: there exists x, y E Z so that ax + by = 1. Bézout’s lemma states that: if P, then...

ADVERTISEMENT
Free Homework Help App
Download From Google Play
Scan Your Homework
to Get Instant Free Answers
Need Online Homework Help?
Ask a Question
Get Answers For Free
Most questions answered within 3 hours.
ADVERTISEMENT
ADVERTISEMENT
ADVERTISEMENT